Aufgabe 5 gelöst

This commit is contained in:
WieErWill 2022-03-05 15:40:32 +01:00
parent 5492ba656d
commit 7340b5b030
2 changed files with 27 additions and 3 deletions

Binary file not shown.

View File

@ -221,11 +221,28 @@ Erlaubte Hilfsmittel: eine math. Formelsammlung/Nachschlagwerk, ein handbeschrie
\begin{parts} \begin{parts}
\part Für welche Partei sollte er sich entscheiden, um mit maximaler Wahrscheinlichkeit Bundeskanzler zu werden? \part Für welche Partei sollte er sich entscheiden, um mit maximaler Wahrscheinlichkeit Bundeskanzler zu werden?
\begin{solution} \begin{solution}
S: wird Spitzenkandidat, K: wird Bundeskanzler,
$P(A \cap S) = 0,1$, $P(B\cap S)=0,2$, $P(C\cap S)=1$
$P(A\cap K)= 0,6$, $P(B\cap K)=0,45$, $P(C\cap K)=0,02$
$P_A(S\cap K) = P(A \cap S) \cap P(A \cap K) = 0,1 * 0,6 = 0,06$
$P_B(S \cap K)= P(B \cap S) \cap P(B \cap K) = 0,2 * 0,45 = 0,09$
$P_C(C \cap K)= P(C \cap S) \cap P(C \cap K)= 1 * 0,02 = 0,02$
Markus hat bei Partei B die größten Chancen Bundeskanzler zu werden (mit 9\%).
\end{solution} \end{solution}
\part Markus lässt die Würfel entscheiden. Bei $1$ tritt er Partei $A$ bei, bei $2$ oder $3$ Partei $B$ und bei $4,5$ oder $6$ gründet er Partei $C$. Markus wird tatsächlich Bundeskanzler. Mit welcher Wahrscheinlichkeit hat er dann Partei $C$ gegründet. \part Markus lässt die Würfel entscheiden. Bei $1$ tritt er Partei $A$ bei, bei $2$ oder $3$ Partei $B$ und bei $4,5$ oder $6$ gründet er Partei $C$. Markus wird tatsächlich Bundeskanzler. Mit welcher Wahrscheinlichkeit hat er dann Partei $C$ gegründet.
\begin{solution} \begin{solution}
$P(\text{Tritt A bei}) = \frac{1}{6}$, $P(\text{Tritt B bei})=\frac{2}{6}=\frac{1}{3}$, $P(\text{Tritt C bei})=\frac{3}{6}=\frac{1}{2}$
$P_{\text{Tritt C bei}}(\text{C gewinnt mit ihm}) = \frac{P_C(C \cap K)}{P(\text{Tritt C bei})} = \frac{0,02}{0,5} = 0,04$
Wenn Markus Bundeskanzler wird, hat er mit 4\% Wahrscheinlichkeit seine eigene Partei C gegründet.
\end{solution} \end{solution}
\end{parts} \end{parts}
@ -315,8 +332,15 @@ Erlaubte Hilfsmittel: eine math. Formelsammlung/Nachschlagwerk, ein handbeschrie
\end{center} \end{center}
\end{solution} \end{solution}
\part Zeige, dass für jede natürliche Zahl $k\leq 1$ gilt: Jeder Baum, der eine Ecke vom Grad $k$ enthält, hat mindestens $k$ Blätter. \part Zeige, dass für jede natürliche Zahl $k\geq 1$ gilt: Jeder Baum, der eine Ecke vom Grad $k$ enthält, hat mindestens $k$ Blätter.
\begin{solution} \begin{solution}
Induktionsannahme: Es wird angenommen der Baum ist homogen verteilt, d.h. die Teilbäume jedes Baumes sind von gleicher Kantenlänge (Größe). Besitzt ein Teilbaum keinen Unterbaum, so ist er ein Blatt.
Induktionsstart: Für $k=1$ besitzt ein Baum maximal $2^1$ Kanten mit mindestens 1 Blatt. Daraus folgt $k=1=\sum Blätter$ stimmt
Induktionsschritt: Für $k=n+1$ besitzt ein Baum maxumal $2^{n+1}$ Kanten mit mindestens 1 Blatt. Daraus folgt $k=n+1...$
\end{solution} \end{solution}
\end{parts} \end{parts}
\end{questions} \end{questions}